LSAT and Law School Admissions Forum

Get expert LSAT preparation and law school admissions advice from PowerScore Test Preparation.

 saffron
  • Posts: 2
  • Joined: Jul 30, 2018
|
#48984
I'm a bit confused about the answer to this question. I guess C technically strengthens the argument but it also seems to disqualify almost every "debating technique" as well. When it come to strengthening the argument questions, is it okay for the correct answer to be this broad, as long as it does something to strengthen the argument?
 James Finch
PowerScore Staff
  • PowerScore Staff
  • Posts: 943
  • Joined: Sep 06, 2017
|
#49073
Hi Saffron,

With Strengthen questions, we have to find the answer choice that will help make the conclusion 100% true/justified. The answer choices are extra premises that will either combine with the existing premises in the stimulus to bolster the conclusion (the correct one) or either do nothing or hurt the argument (incorrect ones). So to test them, we have to plug them into the stimulus and see which one actually strengthens the argument.

(A)--Sounds good at first read, but doesn't actually address the conclusion's scope of avoiding attacks on character, as nothing in the stimulus tells us that anything that reflects an inability to confront an argument means that thing should be avoided. This makes the answer choice irrelevant, and thus wrong.

(B)--Deals with audience reaction, which is clearly irrelevant. Immediate Loser.

(C)--This does a job similar to Justify or Supporter Assumption correct answers, by tying an existing premise to something that is unique to the conclusion. Here, that is tying the avoidance of a tactic (mentioned in the conclusion) with the not confronting an opponent's argument in the premise, making it the correct answer. To make this even clearer, we could conditionalize it:

Relevant :arrow: Avoid

This helps us get to where we want to end up in the conclusion (Avoid) and also ties us to what we already have in the premise (Relevant)

(D)--Deals with the moral right of the person doing the character assassination, which is a reversal of the stimulus's premise. Immediate Loser.

(E)--A Mistaken Negation of (C); what we need is:

Relevant :arrow: Avoid/Raise

But this gives us:

Relevant :arrow: Raise

Hope this clears things up!
 Sambenz
  • Posts: 15
  • Joined: Jun 03, 2020
|
#77539
I ended up choosing E. While I understand why E is wrong, I did not see C as correct either. I understand that we are trying to support the stimulus, but it seems crazy to suggest that the only debating techniques that confront every single argument should be used. Like if a debating technique confronts one or two arguments that is wrong? How does that "principle" truly support what is being stated in the stimulus?
 Jeremy Press
PowerScore Staff
  • PowerScore Staff
  • Posts: 1000
  • Joined: Jun 12, 2017
|
#77898
Hi Sambenz,

The first problem you're having here is that you haven't done what the question stem told you to do: assume that the principles stated in the answer choices are valid ("Which one of the following principles, if valid..."). Sure, answer choice C sounds like a difficult or harsh or unrealistic principle. But you're not supposed to question it. Assume it's valid (i.e. that it's a rule that you should follow).

The only question is whether that rule most helps to justify the conclusion reached in the stimulus, that "[a]ttacks on an opponent’s character should be avoided in political debates." The author says those attacks should be avoided because they do not confront the opponent's argument. I need a rule, no matter how extreme, that covers those bases. The rule from answer choice C covers the bases. If a debate technique that does not confront every argument should not be used, then a debate technique that does not confront any argument (like attacking an opponent's character) would fit the bill and should not be used (because it's a technique that does not confront every argument). So answer choice C most justifies the conclusion. Granted, the rule in answer choice C theoretically prevents you from using a LOT of other techniques as well, but that's fine. I just need a rule (no matter how strong or strict) that covers my stimulus argument. The takeaway is that a Strengthen-Principle answer can be a broader/stronger/harsher rule than would be strictly necessary to get to the conclusion.

I hope this helps!

Jeremy

Get the most out of your LSAT Prep Plus subscription.

Analyze and track your performance with our Testing and Analytics Package.